LSAT and Law School Admissions Forum

Get expert LSAT preparation and law school admissions advice from PowerScore Test Preparation.

User avatar
 Dave Killoran
PowerScore Staff
  • PowerScore Staff
  • Posts: 5852
  • Joined: Mar 25, 2011
|
#85540
Complete Question Explanation
(The complete setup for this game can be found here: lsat/viewtopic.php?f=307&t=9348)

The correct answer choice is (D)

If S is inspected on Monday morning, G cannot be inspected on Monday afternoon. Because G cannot be inspected on Wednesday due to the first rule, G must then be inspected on Tuesday. From the second rule, G cannot be inspected on Tuesday afternoon, and thus G must be inspected on Tuesday morning, and J must be inspected on Tuesday afternoon:

G3-Q17-d1.png

Only L, V, and Z remain to be placed, and as L cannot be inspected on Wednesday due to the first rule, L must be inspected on Monday afternoon:

G3-Q17-d2.png

From the contrapositive of the fourth rule, Z must then be inspected on Wednesday afternoon, leaving V to be inspected on Wednesday morning:

G3-Q17-d3.png

Accordingly, answer choice (D) is correct. Note that the correct answer trades on V, which is the very last variable placed.
You do not have the required permissions to view the files attached to this post.
 maximbasu
  • Posts: 59
  • Joined: May 19, 2016
|
#27554
Hi,

The answer choice, D, doesn't work in this question.

V does not have to be inspected on Wednesday morning.

The setup can look like this:

[M,T,W morning] : S G L
[M,T,W afternoon: V J Z

Please advise. If I'm correct, I just discovered a flaw in the LSAT exam.

Rgs,
Maxim
 Adam Tyson
PowerScore Staff
  • PowerScore Staff
  • Posts: 5153
  • Joined: Apr 14, 2011
|
#27565
Sorry, Maxim, but as much fun as it is to catch the test authors in a mistake, they didn't make one this time. Looks like your hypothetical solution violates the first rule of the game, no hotels on Wednesday. L, a hotel, cannot ever be on Wednesday.

With S on Monday morning, the first thing we know is that G is not on Monday, and is therefore on Tuesday (because hotels cannot be Wednesday). V and Z, our remaining restaurants, must be on Wednesday (again, because hotels cannot be that day). G before J puts J on Tuesday afternoon and G in the morning. L is the leftover and has to go Monday afternoon. What determines the order of V and Z on Wednesday? The contrapositive of the last rule - since L is not in the morning, Z cannot be either.

Take another look, and remember to always check your answers against all of the rules. Some not-laws for G, J and L under the Wednesday group might have been helpful here.

Keep plugging away!
 maximbasu
  • Posts: 59
  • Joined: May 19, 2016
|
#27600
Thanks Adam; you're the best!

I will find a flaw next time.

Maxim
 MichaelR77
  • Posts: 1
  • Joined: Feb 11, 2021
|
#84013
I am unable to answer this question the way the problem is set up.

The problem set-up has an inspector inspect 6 buildings Monday thru Wednesday. 3 Hotels (G, J, L) and 3 Restaurants (S, V, Z).
Exactly 2 buildings are to be inspected per day one in the AM and one in the PM.
Hotels cannot be inspected on Wednesday.
Hotel L and Restaurant Z must be inspected in the AM.
Hotel G must be inspected before Hotel J.
and Hotel G cannot be inspected on the same day as Restaurant S.

Question 17: adds the additional restraint that Restaurant S is inspected Monday AM.
Therefore, Hotel L has to be inspected on Tuesday AM, since it has to be in the morning and cannot be on Wednesday.
Restaurant Z has to be on Wednesday AM, since it also has to be inspected in the morning.
Restaurant V has to be inspected on Wednesday PM, since neither of the other Hotels can be inspected on Wednesday.
This leaves Hotel J and Hotel G to be inspected on Monday PM and Tuesday PM, neither of which work. If Hotel G is inspected on Monday PM, it will then be inspected on the same day as Restaurant S, which violates one of the rules. If Hotel G is inspected on Tuesday PM, it will have to be inspected after Hotel J, thus violating a separate rule..
The problem set-up has an inspector inspect 6 buildings Monday thru Wednesday. 3 Hotels (G, J, L) and 3 Restaurants (S, V, Z).
Exactly 2 buildings are to be inspected per day one in the AM and one in the PM.
Hotels cannot be inspected on Wednesday.
Hotel L and Restaurant Z must be inspected in the AM.
Hotel G must be inspected before Hotel J.
and Hotel G cannot be inspected on the same day as Restaurant S.

Question 17: adds the additional restraint that Restaurant S is inspected Monday AM.
Therefore, Hotel L has to be inspected on Tuesday AM, since it has to be in the morning and cannot be on Wednesday.
Restaurant Z has to be on Wednesday AM, since it also has to be inspected in the morning.
Restaurant V has to be inspected on Wednesday PM, since neither of the other Hotels can be inspected on Wednesday.
This leaves Hotel J and Hotel G to be inspected on Monday PM and Tuesday PM, neither of which work. If Hotel G is inspected on Monday PM, it will then be inspected on the same day as Restaurant S, which violates one of the rules. If Hotel G is inspected on Tuesday PM, it will have to be inspected after Hotel J, thus violating a separate rule.

Now, either I've missed something, or this problem is simply not workable.
 Robert Carroll
PowerScore Staff
  • PowerScore Staff
  • Posts: 1787
  • Joined: Dec 06, 2013
|
#84040
Michael,

I don't see any reason that L and Z have to be inspected in the morning, and those are some of the first statements you made, so I think your faulty inferences come from that.

My thought process was as follows:

The question puts S on Monday morning. I now need to fit all the hotels on Monday afternoon and Tuesday (both morning and afternoon) - three spaces, three variables. So V and Z are on Wednesday in some order.

Looking further, I can't get G on Monday afternoon, because then it would be on the same day as S, but G also has to be earlier than J. The only way to do this is to put G on Tuesday morning, so it can be before J, which just barely fits on Tuesday afternoon. L is now Monday afternoon, its only available spot.

Now that L is on an afternoon, the last rule forces Z on an afternoon as well - Wednesday. V is left, and one space is left - Wednesday morning. That is what answer choice (D) says, so we're done.

Robert Carroll

Get the most out of your LSAT Prep Plus subscription.

Analyze and track your performance with our Testing and Analytics Package.